Added topic tags to some S&J-4 problems + minor typos
[course.git] / latex / problems / Serway_and_Jewett_4 / problem23.06.tex
1 \begin{problem*}{23.6} % transformers
2 A coil of $N=15$ turns and radius $R=10.0\U{cm}$ surrounds a long
3 solenoid of radius $r=2.00\U{cm}$ and $n=1.00\E{3}\U{turns/m}$
4 (Fig.~P23.6).  The current in the solenoid changes as
5 $I=(5.00\U{A})\sin(120t)$.  Find the induced emf in the $15$ turn coil
6 as a function of time.
7 \end{problem*}
8
9 \begin{solution}
10 Because the solenoid is long, we can pretend it is infinite, so all
11 the magnetic field is contained inside the solenoid, and there is no
12 magnetic field outside (see page 751).
13
14 The field inside the solenoid is given by
15 \begin{equation}
16  B = \mu_0 n I \;,
17 \end{equation}
18 so the flux through the large coil is
19 \begin{equation}
20  \Phi_B = \int BdA = N \pi r^2 B = N \pi r^2 \mu_0 n I \;.
21 \end{equation}
22
23 The induced emf is then
24 \begin{equation}
25  \varepsilon = -\frac{d\Phi_B}{dt} = - \pi\mu_0 n N r^2 \frac{dI}{dt}
26    = -\pi\mu_0 n N r^2 (5.00\U{A}\cdot120\U{Hz})\cos(120t)
27    = \ans{-14.2\cos(t \cdot 120\U{rad/s})\U{mV}} \;,
28 \end{equation}
29 where we are assuming that the units on 120 are rad/s, otherwise we'd
30 have to convert them to rad/s to make the units work out on the
31 coefficient.
32 \end{solution}